Mr. Jones starts from rest and begins to accelerate straight to the bathroom at a rate of 0.5 m/s for 10 seconds. What kind of motion is this
A. linear
b centripetal
c.free fall
d projectile​

Answers

Answer 1

Answer:

A. linear

Explanation:

because they are going in a straight line


Related Questions

how does tom and jerry movie character influence your attitude​

Answers

Answer:

it makes me wish I was a cartoon

Answer:

goofy and stupid and act like a kid

Explanation:

branches of sicence​

Answers

Answer: Natural science can be divided into two main branches

Explanation:

life science and physical science. Life science is alternatively known as biology, and physical science is subdivided into branches: physics, chemistry, astronomy and Earth science.

A box of mass 7.0 kg is accelerated from rest across a floor at a rate of 2.0 m/s2 for 9.0 s .Find the net work done on the box. Express your answer to two significant figures and include the appropriate units.

Answers

Answer:

Explanation:

Step one:

given data

mass = 7kg

acceleration =2m/s^2

time= 9seconds

acceleration = velocity/time

velocity= acceleration *time

velocity=2*9

velocity= 18m/s

distance moved= velocity* time

distance= 18*9

distance=162m

we also know that the force on impulse is given as

Ft=mv

F=mv/t

F=7*18/9

F=126/9

F=14N

work done = Force* distance

work done=14*162

work=2268Joules

work= 2.27kJ

38. You are fishing and catch a fish with a mass of
6kg. If the fishing line can withstand a maximum
tension of 30 N, what is the maximum acceleration
you can give the fish as you reel it in?..*
(10 Points)
Enter your answer​

Answers

Answer:

1.7333333m/s²

Explanation:

Tension of the line = the weight + force from pulling up the fish

30N = mg + ma

30 = (6)(9.8) + (6)a

10.4 = 6a

∴ a = 1.7333333m/s²

You are fishing and catch a fish with a mass of 6 kg. If the fishing line can withstand a maximum tension of 30 N, the maximum acceleration is  1.7333333 m/s².

What is acceleration?

The rate at which an item changes its velocity is known as acceleration, a vector quantity. If an object's velocity is changing, it is acceleration.

Tension of the line = the weight + force from pulling up the fish

30 N = mg + ma

30 = (6)(9.8) + (6)a

10.4 = 6 a

a = 1.7333333 m/s²

You are fishing and catch a fish with a mass of 6 kg. If the fishing line can withstand a maximum tension of 30 N, the maximum acceleration is  1.7333333 m/s².

To learn more about acceleration refer to the link:

brainly.com/question/12550364

#SPJ2

A rope is wrapped around the rim of a large uniform solid disk of mass 325 kg and radius 3.00 m. The horizontal disk is made to rotate by pulling on the rope with a constant force of 195 N. If the disk starts from rest, what is its angular speed in rev/s at the end of 2.05 s?

Answers

Answer:

The angular speed is 0.13 rev/s

Explanation:

From the formula

[tex]\tau = I\alpha[/tex]

Where [tex]\tau[/tex] is the torque

[tex]I[/tex] is the moment of inertia

[tex]\alpha[/tex] is the angular acceleration

But, the angular acceleration is given by

[tex]\alpha = \frac{\omega}{t}[/tex]

Where [tex]\omega[/tex] is the angular speed

and [tex]t[/tex] is time

Then, we can write that

[tex]\tau = \frac{I\omega}{t}[/tex]

Hence,

[tex]\omega = \frac{\tau t}{I}[/tex]

Now, to determine the angular speed, we first determine the Torque [tex]\tau[/tex] and the moment of inertia [tex]I[/tex].

Here, The torque is given by,

[tex]\tau = rF[/tex]

Where r is the radius

and F is the force

From the question

r = 3.00 m

F = 195 N

∴ [tex]\tau = 3.00 \times 195[/tex]

[tex]\tau = 585[/tex] Nm

For the moment of inertia,

The moment of inertia of the solid disk is given by

[tex]I = \frac{1}{2}MR^{2}[/tex]

Where M is the mass and

R is the radius

∴[tex]I = \frac{1}{2} \times 325 \times (3.00)^{2}[/tex]

[tex]I = 1462.5[/tex] kgm²

From the question, time t = 2.05 s.

Putting the values into the equation,

[tex]\omega = \frac{\tau t}{I}[/tex]

[tex]\omega = \frac{585 \times 2.05}{1462.5}[/tex]

[tex]\omega = 0.82[/tex] rad/s

Now, we will convert from rad/s to rev/s. To do that, we will divide our answer by 2π

0.82 rad/s = 0.82/2π rev/s

= 0.13 rev/s

Hence, the angular speed is 0.13 rev/s,

A conducting sphere has a net charge of -4.8x10-17 C. What is the approximate number of excess electrons on the sphere?

Answers

Answer:

The number is  [tex]N = 300[/tex]

Explanation:

From the question we are told that

   The  net charge is  [tex]Q = -4.8 *10^{-17 } \ C[/tex]

Generally the charge on a electron is [tex]e = - 1.60 *10^{-19 } \ C[/tex]

Generally the number of excess electrons is mathematically represented as

      [tex]N = \frac{Q}{e}[/tex]

=>  [tex]N = \frac{-4.8 *10^{-17}}{-1.60 *10^{-19}}[/tex]

=>  [tex]N = 300[/tex]

In the video your blood is compared to a __________________ that delivers oxygen to your body and picks up CO2 to be released out when you breath.

Answers

Answer:

delivery truck

Explanation:

because i got it right

Please help!!! I will give brainliest,

Answers

Answer:

C. a liter of salt water.

Explanation:

Defination of Solution =>

a liquid mixture in which the minor component (the solute) is uniformly distributed within the major component (the solvent).

How much would a spring scale with k = 120 N/m stretch, if it had a 3.75 J of work done
on it?

Answers

Answer:

0.25m

Explanation:

Given parameters:

Spring constant , K  = 120N/m

Work done  = 3.75J

Unknown:

magnitude of extension = ?

Solution:

To solve this problem;

           Work done  = [tex]\frac{1}{2}[/tex]kx²  

K is the spring constant

x is the extension

               3.75  =  [tex]\frac{1}{2}[/tex] x 120x²

               3.75  = 60x²

                x²  = 0.06

                x = √0.06  = 0.25m

There are two identical, positively charged conducting spheres fixed in space. The spheres are 40.4 cm apart (center to center) and repel each other with an electrostatic force of F1=0.0720 N . A thin conducting wire connects the spheres, redistributing the charge on each sphere. When the wire is removed, the spheres still repel, but with a force of F2=0.115 N . The Coulomb force constant is k=1/(4π????0)=8.99×109 N⋅m2/C2 . Using this information, find the initial charge on each sphere, q1 and q2 , if q1 is initially less than q2 .

Answers

Answer:

[tex]q_1=5.64\times 10^{-7}\ \text{C}[/tex] and [tex]q_2=2.32\times 10^{-6}\ \text{C}[/tex]

Explanation:

[tex]F_1=0.072\ \text{N}[/tex]

[tex]F_2=0.115\ \text{N}[/tex]

r = Distance between shells = 40.4 cm

[tex]q_1[/tex] and [tex]q_2[/tex] are the charges

[tex]k[/tex] = Coulomb constant = [tex]8.99\times10^{9}\ \text{Nm}^2/\text{C}^2[/tex]

Force is given by

[tex]F_1=\dfrac{kq_1q_2}{r^2}\\\Rightarrow q_1q_2=\dfrac{F_1r^2}{k}\\\Rightarrow q_1q_2=\dfrac{0.072\times 0.404^2}{8.99\times 10^{9}}\\\Rightarrow q_1q_2=1.307\times 10^{-12}\\\Rightarrow q_1=\dfrac{1.307\times 10^{-12}}{q_2}[/tex]

[tex]F_2=\dfrac{kq^2}{r^2}\\\Rightarrow q=\sqrt{\dfrac{F_2r^2}{k}}\\\Rightarrow q=\sqrt{\dfrac{0.115\times 0.404^2}{8.99\times 10^{9}}}\\\Rightarrow q=1.44\times 10^{-6}\ \text{C}[/tex]

[tex]q=\dfrac{q_1+q_2}{2}\\\Rightarrow q_1+q_2=2q\\\Rightarrow q_1+q_2=2\times1.44\times 10^{-6}\\\Rightarrow q_1+q_2=2.88\times 10^{-6}[/tex]

Substituting the above value of [tex]q_1[/tex] we get

[tex]\dfrac{1.307\times 10^{-12}}{q_2}+q_2=2.88\times 10^{-6}\\\Rightarrow q_2^2-2.88\times 10^{-6}q_2+1.307\times 10^{-12}=0\\\Rightarrow \frac{-\left(-0.00000288\right)\pm \sqrt{\left(-0.00000288\right)^2-4\times \:1\times \:1.307\times 10^{-12}}}{2\times \:1}\\\Rightarrow q_2=2.32\times 10^{-6}, 5.64\times 10^{-7}[/tex]

[tex]q_1=\dfrac{1.307\times 10^{-12}}{q_2}=\dfrac{1.307\times 10^{-12}}{2.32\times 10^{-6}}\\\Rightarrow q_1=5.63\times 10^{-7}[/tex]

[tex]q_1=\dfrac{1.307\times 10^{-12}}{q_2}=\dfrac{1.307\times 10^{-12}}{5.64\times 10^{-7}}\\\Rightarrow q_1=2.32\times 10^{-6}[/tex]

Since we know [tex]q_1<q_2[/tex]

[tex]q_1=5.64\times 10^{-7}\ \text{C}[/tex] and [tex]q_2=2.32\times 10^{-6}\ \text{C}[/tex].

A 37.0-kg child swings in a swing supported by two chains, each 3.06 m long. The tension in each chain at the lowest point is 410 N. (a) Find the child's speed at the lowest point.______m/s (b) Find the force exerted by the seat on the child at the lowest point. (Ignore the mass of the seat.)_______ N(upword)

Answers

Answer:

1. 6.15m/s

2. 820N

Explanation:

The total upward force

= 410x2

= 820

g = 9.81

a = v²/r

= 2xT - msg = m x v²/r

= 820-37*9.81 = 37v²/3.06

= 820-362.97 = 37v²/3.06

= 457.03 = 12.09v²

To get v²

V² = 457.03/12.09

V² = 37.8

V = √37.8

V = 6.15m/s

B. We already have the answer to this question

The force exerted is simply gotten by this calculation

2x410

= 820N

Any conclusion reached by analogy is worth accepting

True

False

Answers

that is very much True

A particle with charge q1 C is moving in the positive z-direction at 5 m/s. The magnetic field at its position is B-3 4j1T What is the magnetic force on the particle? A. (20i+15j) N B. (207-15j) N C. (-20i+15j) N D. (-20/-15) N E. none of these

Answers

Answer:

D. [tex]\vec F_{B} = -20\,\hat{i}-15\,\hat{j}\,\,\,[N][/tex]

Explanation:

The statement is not correctly written, the correct form is now described:

A particle with charge [tex]q = -1\,C[/tex] is moving in the positive z-direction at 5 meters per second. The magnetic field at its position is [tex]\vec B = 3\,\hat{i}-4\,\hat{j}\,\,\,[T][/tex]. What is the magnetic force on the particle?

From classic theory on Magnetism, we remember that the magnetic force  exerted on a particle ([tex]\vec F_{B}[/tex]), measured in newtons, is determined by the following vectorial formula:

[tex]\vec F_{B} = q\cdot \vec v \,\times \,\vec B[/tex] (1)

Where:

[tex]q[/tex] - Electric charge, measured in coulombs.

[tex]\vec v[/tex] - Velocity of the particle, measured in meters per second.

[tex]\vec B[/tex] - Magnetic field, measured in teslas.

If we know that [tex]q = -1\,C[/tex], [tex]\vec v = 5\,\hat{k}\,\,\,\left[\frac{m}{s} \right][/tex] and [tex]\vec B = 3\,\hat{i}-4\,\hat{j}\,\,\,[T][/tex], then the magnetic force on the particle is:

[tex]\vec F_{B} = \left|\begin{array}{ccc}\hat{i}&\hat{j}&\hat{k}\\0\,\frac{C\cdot m}{s}&0\,\frac{C\cdot m}{s} &(-1\,C)\cdot (5\,\frac{m}{s} ) \\3\,T&-4\,T&0\,T\end{array}\right|[/tex]

[tex]\vec F_{B} = -(-4\,T)\cdot (-1\,C)\cdot \left(5\,\frac{m}{s} \right)\,\hat{i}+(-1\,C)\cdot\left(5\,\frac{m}{s} \right)\cdot (3\,T)\,\hat{j}[/tex]

[tex]\vec F_{B} = -20\,\hat{i}-15\,\hat{j}\,\,\,[N][/tex]

Which corresponds to option D.

Zinc has a work function of 4.3 eV. a. What is the longest wavelength of light that will release an electron from a zinc surface? b. A 4.7 eV photon strikes the surface and an electron is emitted. What is the maximum possible speed of the electron?

Answers

Answer:

a

[tex]\lambda_{long} = 288.5 \ nm[/tex]

b

The velocity is  [tex]v = 3.7 *0^{5} \ m/s[/tex]

Explanation:

From the question we are told that

   The work function of Zinc is  [tex]W = 4.3 eV[/tex]

Generally the work function can be mathematically represented as

     [tex]E_o = \frac{hc}{\lambda_{long}}[/tex]

=>   [tex]\lambda_{long} = \frac{hc}{E_o}[/tex]

Here  h is the Planck constant with the value  [tex]h = 4.1357 * 10^{-15} eV s[/tex]

  and c is the speed of light with value  [tex]c = 3.0 *10^{8} \ m/s[/tex]

So

     [tex]\lambda_{long} = \frac{4.1357 * 10^{-15} * 3.0 *10^{8}}{4.3}[/tex]

=>  [tex]\lambda_{long} = 2.885 *10^{-7} \ m[/tex]

=>  [tex]\lambda_{long} = 288.5 \ nm[/tex]

Generally the kinetic energy of the emitted electron is mathematically represented as

      [tex]K = E -E_o[/tex]

Here  E is the energy of the photon that strikes the surface

So

    [tex]E- E_o = \frac{1}{2} m * v^2[/tex]

Here m is the mass of electron with value  [tex]m = 9.11*10^{-31 } \ kg[/tex]

Generally  [tex]1 ev = 1.60 *10^{-19} \ J[/tex]

=>   [tex]v = \sqrt{ \frac{2 (E - E_o ) }{ m } }[/tex]

=>    [tex]v = \sqrt{ \frac{2 (4.7 - 4.3 )* 1.60 *10^{-19} }{ 9.11 *10^{-31} } }[/tex]

=>    [tex]v = 3.7 *0^{5} \ m/s[/tex]

   

A guitar string produces 3 beats/s when sounded with a 352 Hz tuning fork and 8 beat/s when sounded with a 357 Hz tuning fork. What is the vibrational frequency (in Hz) of the string

Answers

Answer:

349 Hz

Explanation:

We are told that the guitar string produces 3 beats/s when sounded with a 352 Hz tuning fork and 8 beat/s when sounded with a 357 Hz tuning fork.

This means that for the 352 Hz tuning fork, the vibrational frequency is;

f = 352 ± 3

f = (352 + 3) or (352 - 3)

f = 355 Hz or 349Hz

For the 357 Hz tuning fork, the vibrational frequency is;

f = 357 ± 8

f = (357 + 8) or (357 - 8)

f = 365 Hz or 349 Hz

In both cases, 349 Hz is common;

Thus, the vibrational frequency of the string = 349 Hz

A bullet is fired horizontally at a height of 2 meters at a velocity of 930 m/s. Assume no air resistance. How long until the bullet reaches the ground?


0.32 s
0.57 s
0.64 s
0.25 s

Answers

Should be 0.64 seconds

During a circus performance, a 72-kg humancannonball is shot out of an 18-m-long cannon. If thehuman cannonball spends 0.95 s in the cannon,determine the average net force exerted on him in thebarrel of the cannon.

Answers

Answer:

2872.8 N

Explanation:

We have the following information

m =n72kg

Δy = 18m

t = 0.95s.

From here we use the equation

Δy=1/2at2 in order to solve for the acceleration.

So a

=( 2x 18m)/(0.95s²)

= 36/0.9025

= 39.9m/s2.

From there we use the equation

F = ma

F=(72kg) x (39.9)

= 2872.8N.

2872.8N is the average net force exerted on him in the barrel of the cannon.

Thank you!

Describe why you are doing the experimen?​

Answers

What is the experiment ?

Answer:

An experiment is a procedure carried out to support, refute, or validate a hypothesis. Experiments provide insight into cause-and-effect by demonstrating what outcome occurs when a particular factor is manipulated.

Explanation:

Two particles are separated by 0.38 m and have charges of -6.25 x 10-9C
and 2.91 x 10-9 C. Use Coulomb's law to predict the force between the
particles if the distance is doubled. The equation for Coulomb's law is
Fe = kq92, and the constant, k, equals 9.00 x 10°N-m/c2.
A. -2.83 x 10-7N
B. 2.83 x 10-7N
C. -1.13 x 10-6N
D. 1.13 x 10-6N

Answers

Answer:A

Explanation:

Answer:

A. -2.83 x 10-7N

Explanation:

Block A is also connected to a horizontally-mounted spring with a spring constant of 281 J/m2. What is the angular frequency (in rad/s) of simple harmonic oscillations of this system?

Answers

Answer:

This question is incomplete

Explanation:

This question is incomplete. However, the formula to be used here is

ω = 2π/T

Where ω is the angular frequency (in rad/s)

T is the period - the time taken for Block A to complete one oscillation and return to it's original position.

To solve for this period T, the formula below should be used

T = 2π√m/k

where m is the mass of the object (Block A) and k is the spring constant (281 J/m²)

Suppose a certain object has a mass of 5.00 kilograms on the earth. On the
Moon, where g is 1.6 m/s/s what would its mass be?*

Answers

Answer:

it would be 49.03325 Newton.

What is the result of increasing the speed at which a magnet moves in and
out of a wire coil?
A. The current in the wire increases.
B. The magnetic field around the magnet decreases.
C. The current in the wire decreases.
D. The magnetic field around the magnet increases.

Answers

Answer:

A. The current in the wire increases.

Explanation:

Increasing the speed at which a magnet moves in and out of a wire coil increases the current in the wire.

This phenomenon shows the inter-relationship between electricity and magnetic fields.

Magnetic fields are induced by passage of electric current. Also, electric current can be produce by magnetic fields. When the speed at which a magnet moves in and out of a wire coil increases, the current also increases.

Answered: A 4 kg mass is attached to a horizontal spring with the spring constant of 600 N/m and rests on a frictionless surface on the ground. The spring is compressed 0.5 m past its equilibrium. What is the initial energy of the system.

Answer: 75 joules

Answers

glad it’s figured out

Can someone help me with my physics

A pendulum is a body that is suspended from a fixed point so that it can swing back and forth through an exchange of kinetic energy and gravitational potential energy. Using 1–2 sentences, explain what happens to the kinetic energy and gravitational potential energy of the pendulum at the highest point and at the lowest point of its swing.

Answers

In a simple pendulum with no friction, mechanical energy is conserved. Total mechanical energy is a combination of kinetic energy and gravitational potential energy. As the pendulum swings back and forth, there is a constant exchange between kinetic energy and gravitational potential energy.

Answer:

Because mechanical energy (the sum of potential and kinetic energy) is conserved, as the kinetic energy increases, the potential energy decreases. The maximum kinetic energy is achieved when the pendulum passes through the lowest point, and the maximum potential energy is achieved at the highest point.

Resistor is made of a very thin metal wire that is 3.2 mm long, with a diameter of 0.4 mm. What is the electric field inside this metal resistor?

Answers

Complete question:

Resistor is made of a very thin metal wire that is 3.2 mm long, with a diameter of 0.4 mm. What is the electric field inside this metal resistor? If the potential difference due to electric field between the two ends of the resistor is 10 V.

Answer:

The electric field inside this metal resistor is 3125 V/m

Explanation:

Given;

length of the wire, L = 3.2 mm = 3.2 x 10⁻³ m

diameter of the wire, d = 0.4 mm = 0.4 x 10⁻³ m

the potential difference due to electric field between the two ends of the resistor, V = 10 V

The electric field inside this metal resistor is given by;

ΔV = EL

where;

ΔV is change in electric potential

E = ΔV / L

E = 10 / (3.2 x 10⁻³ )

E = 3125 V/m

Therefore, the electric field inside this metal resistor is 3125 V/m

what can i yeet baby or toddler

Answers

Answer:

both

Explanation:

baby for fun, toddler for vengence

A 5.00-kg object is attached to one end of a horizontal spring that has a negligible mass and a spring constant of 280 N/m. The other end of the spring is fixed to a wall. The spring is compressed by 10.0 cm from its equilibrium position and released from rest.
1) What is the speed of the object when it is 8.00 cm from equilibrium? (Express your answer to three significant figures.)
2) What is the speed when the object is 5.00 cm from equilibrium? (Express your answer to three significant figures.)
3) What is the speed when the object is at the equilibrium position? (Express your answer to three significant figures.)

Answers

Answer:

1) v = 0.45 m/s

2) v = 0.65 m/s

3) v = 0.75 m/s  

Explanation:

1) We can find the speed of the object by conservation of energy:

[tex] E_{i} = E_{f} [/tex]

[tex] \frac{1}{2}kx^{2} = \frac{1}{2}kx^{2} + \frac{1}{2}mv^{2} [/tex]

Where:

k: is the spring constant = 280 N/m

v: is the speed of the object =?

m: is the mass of the object = 5.00 kg

x: is the displacement of the spring

[tex] \frac{1}{2}280N/m(0.10 m)^{2} = \frac{1}{2}280N/m(0.08 m)^{2} + \frac{1}{2}5.00 kgv^{2} [/tex]                              

[tex] v = \sqrt{\frac{280N/m(0.10 m)^{2} - 280N/m(0.08 m)^{2}}{5.00 kg}} = 0.45 m/s [/tex]

2) When the object is 5.00 cm (0.050 m) from equilibrium, the speed of the object is:

[tex] \frac{1}{2}kx^{2} = \frac{1}{2}kx^{2} + \frac{1}{2}mv^{2} [/tex]    

[tex] \frac{1}{2}280N/m(0.10 m)^{2} = \frac{1}{2}280N/m(0.05 m)^{2} + \frac{1}{2}5.00 kgv^{2} [/tex]      

[tex] v = \sqrt{\frac{280N/m(0.10 m)^{2} - 280N/m(0.05 m)^{2}}{5.00 kg}} = 0.65 m/s [/tex]      

 

3) When the object is at the equilibrium position, the speed of the object is:

[tex] \frac{1}{2}kx^{2} = \frac{1}{2}kx^{2} + \frac{1}{2}mv^{2} [/tex]    

[tex] \frac{1}{2}280N/m(0.10 m)^{2} = \frac{1}{2}280N/m(0 m)^{2} + \frac{1}{2}5.00 kgv^{2} [/tex]      

[tex] v = \sqrt{\frac{280N/m(0.10 m)^{2}}{5.00 kg}} = 0.75 m/s [/tex]

I hope it helps you!                                                                                        

(1) the speed of the object when compression of the spring is 8 cm is 0.449 m/s

(2) the speed of the object when compression of the spring is 5 cm is 0.648 m/s

(3) the speed of the object when the spring is at equilibrium is 0.748 m/s

Compression of spring and conservation of energy:

Given that the mass of the object, m = 5 kg

spring constant, k = 280 N/m

compression of the spring , x = 10 cm = 0.1m

(i) the spring compression is at d = 8cm

according to the conservation of energy:

[tex]\frac{1}{2}kx^2=\frac{1}{2}kd^2+\frac{1}{2}mv^2[/tex]

where v is the speed at the given compression of the spring.

[tex]\frac{1}{2}\times280\times(0.1)^2=\frac{1}{2}\times280\times(0.08)^2+\frac{1}{2}\times5\times v^2\\\\v^2=0.2016[/tex]

v = 0.449 m/s

(ii) the spring compression is at d = 5cm

according to the conservation of energy:

[tex]\frac{1}{2}kx^2=\frac{1}{2}kd^2+\frac{1}{2}mv^2[/tex]

where v is the speed at the given compression of the spring.

[tex]\frac{1}{2}\times280\times(0.1)^2=\frac{1}{2}\times280\times(0.05)^2+\frac{1}{2}\times5\times v^2\\\\v^2=0.42[/tex]

v = 0.648 m/s

(iii) the spring is at equilibrium so compression is at d = 0cm

according to the conservation of energy:

[tex]\frac{1}{2}kx^2=\frac{1}{2}kd^2+\frac{1}{2}mv^2[/tex]

where v is the speed at the given compression of the spring.

[tex]\frac{1}{2}\times280\times(0.1)^2=\frac{1}{2}\times280\times(0)^2+\frac{1}{2}\times5\times v^2\\\\v^2=0.56[/tex]

v = 0.748 m/s

Learn more about conservation of energy:

https://brainly.com/question/14245799?referrer=searchResults

7. What does the changing colour perceived by the person as the filter changes indicate to you
about white light?

Answers

Answer:

lo

Explanation:

What becomes V if we use 2 resistors of 4W in parallel?
A. 2.66 V
B. 6 V
C. 12 V
D. 24 V

Answers

Answer:

This question is incomplete.

Explanation:

This question is incomplete. However, it should be noted that the voltage, V, across resistors in parallel is the same (although there currents are not the same). Thus, if a voltage has been provided, it remains the same but if not provided, you can solve for it using the formulas below

V = IR

where V is the voltage. I is the current and R is the resistance

R in parallel can be calculated as R = 1/R₁ + 1/R₂ + 1/R₃ + ......

A train is traveling at 55m/s begins to slow down as it approaches a bend in the tracks. If it travels around the bend at a speed of9 m/s and it takes 49 s to properly slow down, what distance does the train travel while slowing down?

Answers

Answer:

x = 1127 [m]

Explanation:

In order to solve this problem, we must use the equations of kinematics. With the first equation, we must find the acceleration and with the second equation we must find the distance.

[tex]v_{f} =v_{o} -a*t[/tex]

where:

Vf = final velocity = 9 [m/s]

Vo = initial velocity = 55 [m/s]

a = acceleration o desacceleration [m/s²]

t = time = 49 [s]

Now replacing:

9 = 55 - a*49

a*49 = 55 + 9

a = 1.306 [m/s²]

Note: The negative sign in the above equation means that the speed decreases.

Now using the second equation.

[tex]v_{f}^{2} =v_{o}^{2} -2*a*x[/tex]

(9)² = (55)² - 2*(1.306)*x

2944 = 2.612*x

x = 1127 [m]

Other Questions
(04.01 LC)The coordinates of point A on a grid are (-2,-4). Point A is reflected across the y-axis to obtain point B. The coordinates of point B are- 4). (5 points) Draw conclusion: The density of water is 1.0 g/mL, or 1.0 g/cm. Look at the data in your table. How can you use the density of an object to predict whether it will sink or float? (Data is above)PLEASE HELP A water balloon is thrown upward from a height of 5 feet with an initial velocity of 35 feet per second. The quadratic function h(t)=-16t^2+35t+5 represents the height of the balloon, h, in feet t seconds after it is thrown. Determine when the balloon is more than 10 feet above the ground. Round your answer to the nearest thousandth. Francesca obtains a loan of $18.965.00. The loan has a simple interest rate of 3%. How much interest will Francesca owe at the end of the first year? Given m||n, find the value of x. What value of b will cause the system to have aninfinite number of solutions?6y = 6x-b+3x+12y=-3-3x + y = -3b = PLEASE NEED HELP URGENTLY!Raul tried to find the midpoint between 5 & 3i and 12 + 7i. Check his work below.Sum=(5+(-3))+(12+7)i=2+19i so the midpoint is 1+ 19over2 iWhat mistake did Raul make?AHe subtracted the values instead of adding them.BHe did not add the corresponding parts of the complex numbers.CHe found the sum instead of the average because he forgot to divide by two.DHe added the numbers in the wrong order. For Spanish language or whatever1. What part of speech does an adjective describe? 2. What parts of speech are described by adverbs? The diameter of a proton is 1.7 x 10-15 meters and the diameter of an atom is 5 x 10-10 meters. What is the difference in the diameters? Write your answer in scientific notation. How do doctors use hemoglobin levels to decidewhether or not someone was blood doping? PLEASE HELP ME! I need help on this question If the ratio of the sides of two similar prisms is 4:1. Is the ratio of the volume of the prisms the same? (4:1) Imagine that a sound engineer has been asked to produce the sound of a dog barking. Why would the sound engineer need to know the size of the dog and the circumstances to make it believable?Most people can tell the breed of dog by the bark.Small dogs have lower, deeper barks than large dogs.Creating a dog bark sound is not the job of a sound engineer.These factors would determine the tone and intensity of the bark. what important things about physical fitness can your resting Target and recovery heart rates tell you Explain at least 2 steps a person can take to develop or keep up a positive body image. Read the excerpt from A Girl from Yamhill.He does not faint. Somehow Mother boosts him along to the parlor couch. Later, after the doctor has gone, I learn that a sudden .... on the reins by a team of horses has dislocated the arm, an accident that has happened before in his heavy farm work, for his shoulder sockets are too shallow for the weight of his muscles. Mothers determination always supports him to the couch.Which universal theme is present in this excerpt?Hard work often leads to injuries and suffering.People sacrifice and work hard for those whom they love.People who sacrifice for their families are rewarded.A person who works too hard is forced to depend on others. You randomly select one truck owner. What is the probability that he owns a Dodge or has fourwheel drive? calling sweden what is not a factor contributing to the rise of export activity in the word economy g Three-sevenths of the students in a class play a sport, and, of these students, one-third play soccer. What fraction of the class plays soccer? Please, I really need help! BRAINLIST IF ANSWER!!What is the theme in Love that dog by Sharon Creech?There is a pdf of the story attached to my questionTHANK YOU :)